How prove this $(p-1)!left(1+frac12+frac13+cdots+frac1p-1right)equiv 0pmodp^2$ The Next CEO of Stack OverflowA proof of Wolstenholme's theoremWhy is this number always divisible by $p^2$Prove that $x^2 equiv a pmod2^e$ is solvable $forall e$ iff $a equiv 1 pmod8$How can prove this $sum_k=0^pbinompk(pm isqrt3)^kequiv 1pm (isqrt3)^p-psum_k=1^p-1frac(mp isqrt3)^kkpmodp^2?$Prove $a^pq equiv a pmod pq$How prove this $binomnmequiv 0pmod p$Prove that if $p>2$ is prime then $left(prod_k=1^p-1k^kright)^2equiv (-1)^fracp+12pmod p.$proving $1 + a + a^2 + cdots + a^h-1 equiv 0 pmodp$.How prove this diophantine equation $x^2-y^2equiv apmod p$ have only $p-1$rootsProve that if $ab equiv cd pmodn$ and $b equiv d pmod n$ and $gcd(b, n) = 1$ then $a equiv c pmod n$.$left(frac3p right)=1$ iff $pequiv 1pmod12$ or $pequiv -1pmod12$Prove $left(frac-3pright)=1$ if $pequiv 1 pmod 3$

Is it ok to trim down a tube patch?

0-rank tensor vs vector in 1D

Could a dragon use its wings to swim?

My ex-girlfriend uses my Apple ID to login to her iPad, do I have to give her my Apple ID password to reset it?

Defamation due to breach of confidentiality

From jafe to El-Guest

Touchpad not working on Debian 9

Which one is the true statement?

Traduction de « Life is a roller coaster »

Does the Idaho Potato Commission associate potato skins with healthy eating?

Is there a difference between "Fahrstuhl" and "Aufzug"?

Is it okay to majorly distort historical facts while writing a fiction story?

Easy to read palindrome checker

Can I calculate next year's exemptions based on this year's refund/amount owed?

Is there a way to save my career from absolute disaster?

Is there such a thing as a proper verb, like a proper noun?

Help! I cannot understand this game’s notations!

Small nick on power cord from an electric alarm clock, and copper wiring exposed but intact

Why doesn't UK go for the same deal Japan has with EU to resolve Brexit?

Is it convenient to ask the journal's editor for two additional days to complete a review?

How to Implement Deterministic Encryption Safely in .NET

Is a distribution that is normal, but highly skewed, considered Gaussian?

Graph of the history of databases

A question about free fall, velocity, and the height of an object.



How prove this $(p-1)!left(1+frac12+frac13+cdots+frac1p-1right)equiv 0pmodp^2$



The Next CEO of Stack OverflowA proof of Wolstenholme's theoremWhy is this number always divisible by $p^2$Prove that $x^2 equiv a pmod2^e$ is solvable $forall e$ iff $a equiv 1 pmod8$How can prove this $sum_k=0^pbinompk(pm isqrt3)^kequiv 1pm (isqrt3)^p-psum_k=1^p-1frac(mp isqrt3)^kkpmodp^2?$Prove $a^pq equiv a pmod pq$How prove this $binomnmequiv 0pmod p$Prove that if $p>2$ is prime then $left(prod_k=1^p-1k^kright)^2equiv (-1)^fracp+12pmod p.$proving $1 + a + a^2 + cdots + a^h-1 equiv 0 pmodp$.How prove this diophantine equation $x^2-y^2equiv apmod p$ have only $p-1$rootsProve that if $ab equiv cd pmodn$ and $b equiv d pmod n$ and $gcd(b, n) = 1$ then $a equiv c pmod n$.$left(frac3p right)=1$ iff $pequiv 1pmod12$ or $pequiv -1pmod12$Prove $left(frac-3pright)=1$ if $pequiv 1 pmod 3$










9












$begingroup$



Show that
$$(p-1)!left(1+dfrac12+dfrac13+cdots+dfrac1p-1right)equiv 0pmodp^2.$$




Maybe use this
$$dfrac1k+dfrac1p-k=dfracpk(p-k)$$
and then I can't. Can you help me to prove it?



Thank you.










share|cite|improve this question











$endgroup$







  • 1




    $begingroup$
    Adapting Wilson's theorem mod $p^2$?
    $endgroup$
    – PITTALUGA
    Nov 6 '13 at 12:09











  • $begingroup$
    I think the general expression is not an integer; at least for p=3,5.
    $endgroup$
    – user99680
    Nov 6 '13 at 12:15






  • 1




    $begingroup$
    It is, since every denominator appears as a factor of $(p-1)!$.
    $endgroup$
    – Christoph
    Nov 6 '13 at 12:16







  • 2




    $begingroup$
    More is true: en.wikipedia.org/wiki/Wolstenholme%27s_theorem.
    $endgroup$
    – lhf
    Nov 6 '13 at 12:17










  • $begingroup$
    Ah, yes, I was adding incorrectly.
    $endgroup$
    – user99680
    Nov 6 '13 at 12:18















9












$begingroup$



Show that
$$(p-1)!left(1+dfrac12+dfrac13+cdots+dfrac1p-1right)equiv 0pmodp^2.$$




Maybe use this
$$dfrac1k+dfrac1p-k=dfracpk(p-k)$$
and then I can't. Can you help me to prove it?



Thank you.










share|cite|improve this question











$endgroup$







  • 1




    $begingroup$
    Adapting Wilson's theorem mod $p^2$?
    $endgroup$
    – PITTALUGA
    Nov 6 '13 at 12:09











  • $begingroup$
    I think the general expression is not an integer; at least for p=3,5.
    $endgroup$
    – user99680
    Nov 6 '13 at 12:15






  • 1




    $begingroup$
    It is, since every denominator appears as a factor of $(p-1)!$.
    $endgroup$
    – Christoph
    Nov 6 '13 at 12:16







  • 2




    $begingroup$
    More is true: en.wikipedia.org/wiki/Wolstenholme%27s_theorem.
    $endgroup$
    – lhf
    Nov 6 '13 at 12:17










  • $begingroup$
    Ah, yes, I was adding incorrectly.
    $endgroup$
    – user99680
    Nov 6 '13 at 12:18













9












9








9


4



$begingroup$



Show that
$$(p-1)!left(1+dfrac12+dfrac13+cdots+dfrac1p-1right)equiv 0pmodp^2.$$




Maybe use this
$$dfrac1k+dfrac1p-k=dfracpk(p-k)$$
and then I can't. Can you help me to prove it?



Thank you.










share|cite|improve this question











$endgroup$





Show that
$$(p-1)!left(1+dfrac12+dfrac13+cdots+dfrac1p-1right)equiv 0pmodp^2.$$




Maybe use this
$$dfrac1k+dfrac1p-k=dfracpk(p-k)$$
and then I can't. Can you help me to prove it?



Thank you.







elementary-number-theory






share|cite|improve this question















share|cite|improve this question













share|cite|improve this question




share|cite|improve this question








edited Mar 15 '16 at 10:37









user26857

39.5k124283




39.5k124283










asked Nov 6 '13 at 12:04







user94270














  • 1




    $begingroup$
    Adapting Wilson's theorem mod $p^2$?
    $endgroup$
    – PITTALUGA
    Nov 6 '13 at 12:09











  • $begingroup$
    I think the general expression is not an integer; at least for p=3,5.
    $endgroup$
    – user99680
    Nov 6 '13 at 12:15






  • 1




    $begingroup$
    It is, since every denominator appears as a factor of $(p-1)!$.
    $endgroup$
    – Christoph
    Nov 6 '13 at 12:16







  • 2




    $begingroup$
    More is true: en.wikipedia.org/wiki/Wolstenholme%27s_theorem.
    $endgroup$
    – lhf
    Nov 6 '13 at 12:17










  • $begingroup$
    Ah, yes, I was adding incorrectly.
    $endgroup$
    – user99680
    Nov 6 '13 at 12:18












  • 1




    $begingroup$
    Adapting Wilson's theorem mod $p^2$?
    $endgroup$
    – PITTALUGA
    Nov 6 '13 at 12:09











  • $begingroup$
    I think the general expression is not an integer; at least for p=3,5.
    $endgroup$
    – user99680
    Nov 6 '13 at 12:15






  • 1




    $begingroup$
    It is, since every denominator appears as a factor of $(p-1)!$.
    $endgroup$
    – Christoph
    Nov 6 '13 at 12:16







  • 2




    $begingroup$
    More is true: en.wikipedia.org/wiki/Wolstenholme%27s_theorem.
    $endgroup$
    – lhf
    Nov 6 '13 at 12:17










  • $begingroup$
    Ah, yes, I was adding incorrectly.
    $endgroup$
    – user99680
    Nov 6 '13 at 12:18







1




1




$begingroup$
Adapting Wilson's theorem mod $p^2$?
$endgroup$
– PITTALUGA
Nov 6 '13 at 12:09





$begingroup$
Adapting Wilson's theorem mod $p^2$?
$endgroup$
– PITTALUGA
Nov 6 '13 at 12:09













$begingroup$
I think the general expression is not an integer; at least for p=3,5.
$endgroup$
– user99680
Nov 6 '13 at 12:15




$begingroup$
I think the general expression is not an integer; at least for p=3,5.
$endgroup$
– user99680
Nov 6 '13 at 12:15




1




1




$begingroup$
It is, since every denominator appears as a factor of $(p-1)!$.
$endgroup$
– Christoph
Nov 6 '13 at 12:16





$begingroup$
It is, since every denominator appears as a factor of $(p-1)!$.
$endgroup$
– Christoph
Nov 6 '13 at 12:16





2




2




$begingroup$
More is true: en.wikipedia.org/wiki/Wolstenholme%27s_theorem.
$endgroup$
– lhf
Nov 6 '13 at 12:17




$begingroup$
More is true: en.wikipedia.org/wiki/Wolstenholme%27s_theorem.
$endgroup$
– lhf
Nov 6 '13 at 12:17












$begingroup$
Ah, yes, I was adding incorrectly.
$endgroup$
– user99680
Nov 6 '13 at 12:18




$begingroup$
Ah, yes, I was adding incorrectly.
$endgroup$
– user99680
Nov 6 '13 at 12:18










3 Answers
3






active

oldest

votes


















6












$begingroup$

The solution below is adapted from Notes on Wolstenholme’s Theorem by Timothy H. Choi.



Let
$$
S=(p-1)!sum_k=1^p-1 frac1k
$$
Using your insight
$$
dfrac1k+dfrac1p-k=dfracpk(p-k)
$$
we have
$$
2S=(p-1)!sum_k=1^p-1 left(dfrac1k+dfrac1p-kright) =
psum_k=1^p-1 frac(p-1)!k(p-k) = pS'
$$
Note that $S'$ is an integer. Now
$$
frac(p-1)!k(p-k) equiv (k^2)^-1 bmod p
$$
where the inverse is taken $bmod p$. This is a consequence of Wilson’s Theorem.
Hence
$$
S'equiv
sum_k=1^p-1 (k^2)^-1 equiv
sum_k=1^p-1 k^2 = frac(p-1)p(2(p-1)+1)6 equiv 0 bmod p
$$
This means that $2Sequiv 0 bmod p^2$ and so $Sequiv 0 bmod p^2$. (We need $p>3$ twice here.)






share|cite|improve this answer











$endgroup$




















    4












    $begingroup$

    As others have noted, the congruence is not true for $p=3$, since
    $$ 2!left(1+frac 1 2right)=2+1=3,$$
    which is not divisible by $9$. We can still use what you suggested to prove the congruence holds $operatornamemod p$. Let $p$ be an odd prime, then
    beginalign*
    (p-1)!sum_k=1^p-1 frac 1 k &= (p-1)! sum_k=1^(p-1)/2 left(frac 1 k + frac 1 p-kright) \&= (p-1)!sum_k=1^(p-1)/2 fracpk(p-k) = psum_k=1^(p-1)/2 frac(p-1)!k(p-k).
    endalign*
    Since $(p-1)!$ always contains $k$ and $(p-k)$ as a factor, the fractions in the sum are integers and the result is a multiple of $p$.



    See lhf's answer for why it is even a multiple of $p^2$ as long as $p>3$.






    share|cite|improve this answer











    $endgroup$












    • $begingroup$
      The congruence will hold for any $p>3$ modulo $p^2$. That's Wolstenholme's theorem as lhf pointed out.
      $endgroup$
      – EuYu
      Nov 6 '13 at 12:39






    • 1




      $begingroup$
      I adapted my post and referencered lhf's answer, thanks for your comment!
      $endgroup$
      – Christoph
      Nov 6 '13 at 12:54


















    0












    $begingroup$

    I came up with this proof while I work on this problem.



    We write $f(x) equiv_p g(x)$ if polynomials $f(x), g(x) in mathbbZ[x]$ satisfies $a_iequiv b_i$ mod $p$ for all coefficients $a_i$ of $f(x)$, and $b_i$ of $g(x)$.



    Then we have by Fermat's theorem,
    $$
    x^p-1-1 equiv_p (x-1)(x-2) cdots (x-(p-1)).
    $$



    Group the numbers in pairs as discussed here already,
    $$
    frac 11+frac 1p-1=frac p1(p-1), frac12+ frac 1p-2=frac p2(p-2), cdots, frac 1(p-1)/2+frac1(p+1)/2=frac p (p-1)(p+1)/4 .
    $$

    Each group has numerator divisible by $p$. Then it suffices to prove that the numerator $N$ in
    $$
    sum_j=1^(p-1)/2 frac 1j(p-j)=frac N(p-1)! (*)
    $$

    is divisible by $p$.



    We may write



    beginalign*x^p-1-1&equiv_p
    (x-1)(x-2)cdots (x-(p-1))\ &= prod_j=1^(p-1)/2 (x-j)(x-(p-j)) \ &=prod_j=1^(p-1)/2 (x^2-(j+(p-j))x + j(p-j))\
    &equiv_pprod_j=1^(p-1)/2 (x^2+j(p-j))
    endalign*

    Expanding the last product, we find that $N$ is the coefficient of $x^2$. Since $pgeq 5$, the coefficient of $x^2$ in the product must be divisible by $p$. Hence, $p|N$ as desired.






    share|cite|improve this answer









    $endgroup$













      Your Answer





      StackExchange.ifUsing("editor", function ()
      return StackExchange.using("mathjaxEditing", function ()
      StackExchange.MarkdownEditor.creationCallbacks.add(function (editor, postfix)
      StackExchange.mathjaxEditing.prepareWmdForMathJax(editor, postfix, [["$", "$"], ["\\(","\\)"]]);
      );
      );
      , "mathjax-editing");

      StackExchange.ready(function()
      var channelOptions =
      tags: "".split(" "),
      id: "69"
      ;
      initTagRenderer("".split(" "), "".split(" "), channelOptions);

      StackExchange.using("externalEditor", function()
      // Have to fire editor after snippets, if snippets enabled
      if (StackExchange.settings.snippets.snippetsEnabled)
      StackExchange.using("snippets", function()
      createEditor();
      );

      else
      createEditor();

      );

      function createEditor()
      StackExchange.prepareEditor(
      heartbeatType: 'answer',
      autoActivateHeartbeat: false,
      convertImagesToLinks: true,
      noModals: true,
      showLowRepImageUploadWarning: true,
      reputationToPostImages: 10,
      bindNavPrevention: true,
      postfix: "",
      imageUploader:
      brandingHtml: "Powered by u003ca class="icon-imgur-white" href="https://imgur.com/"u003eu003c/au003e",
      contentPolicyHtml: "User contributions licensed under u003ca href="https://creativecommons.org/licenses/by-sa/3.0/"u003ecc by-sa 3.0 with attribution requiredu003c/au003e u003ca href="https://stackoverflow.com/legal/content-policy"u003e(content policy)u003c/au003e",
      allowUrls: true
      ,
      noCode: true, onDemand: true,
      discardSelector: ".discard-answer"
      ,immediatelyShowMarkdownHelp:true
      );



      );













      draft saved

      draft discarded


















      StackExchange.ready(
      function ()
      StackExchange.openid.initPostLogin('.new-post-login', 'https%3a%2f%2fmath.stackexchange.com%2fquestions%2f554261%2fhow-prove-this-p-1-left1-frac12-frac13-cdots-frac1p-1-right%23new-answer', 'question_page');

      );

      Post as a guest















      Required, but never shown
























      3 Answers
      3






      active

      oldest

      votes








      3 Answers
      3






      active

      oldest

      votes









      active

      oldest

      votes






      active

      oldest

      votes









      6












      $begingroup$

      The solution below is adapted from Notes on Wolstenholme’s Theorem by Timothy H. Choi.



      Let
      $$
      S=(p-1)!sum_k=1^p-1 frac1k
      $$
      Using your insight
      $$
      dfrac1k+dfrac1p-k=dfracpk(p-k)
      $$
      we have
      $$
      2S=(p-1)!sum_k=1^p-1 left(dfrac1k+dfrac1p-kright) =
      psum_k=1^p-1 frac(p-1)!k(p-k) = pS'
      $$
      Note that $S'$ is an integer. Now
      $$
      frac(p-1)!k(p-k) equiv (k^2)^-1 bmod p
      $$
      where the inverse is taken $bmod p$. This is a consequence of Wilson’s Theorem.
      Hence
      $$
      S'equiv
      sum_k=1^p-1 (k^2)^-1 equiv
      sum_k=1^p-1 k^2 = frac(p-1)p(2(p-1)+1)6 equiv 0 bmod p
      $$
      This means that $2Sequiv 0 bmod p^2$ and so $Sequiv 0 bmod p^2$. (We need $p>3$ twice here.)






      share|cite|improve this answer











      $endgroup$

















        6












        $begingroup$

        The solution below is adapted from Notes on Wolstenholme’s Theorem by Timothy H. Choi.



        Let
        $$
        S=(p-1)!sum_k=1^p-1 frac1k
        $$
        Using your insight
        $$
        dfrac1k+dfrac1p-k=dfracpk(p-k)
        $$
        we have
        $$
        2S=(p-1)!sum_k=1^p-1 left(dfrac1k+dfrac1p-kright) =
        psum_k=1^p-1 frac(p-1)!k(p-k) = pS'
        $$
        Note that $S'$ is an integer. Now
        $$
        frac(p-1)!k(p-k) equiv (k^2)^-1 bmod p
        $$
        where the inverse is taken $bmod p$. This is a consequence of Wilson’s Theorem.
        Hence
        $$
        S'equiv
        sum_k=1^p-1 (k^2)^-1 equiv
        sum_k=1^p-1 k^2 = frac(p-1)p(2(p-1)+1)6 equiv 0 bmod p
        $$
        This means that $2Sequiv 0 bmod p^2$ and so $Sequiv 0 bmod p^2$. (We need $p>3$ twice here.)






        share|cite|improve this answer











        $endgroup$















          6












          6








          6





          $begingroup$

          The solution below is adapted from Notes on Wolstenholme’s Theorem by Timothy H. Choi.



          Let
          $$
          S=(p-1)!sum_k=1^p-1 frac1k
          $$
          Using your insight
          $$
          dfrac1k+dfrac1p-k=dfracpk(p-k)
          $$
          we have
          $$
          2S=(p-1)!sum_k=1^p-1 left(dfrac1k+dfrac1p-kright) =
          psum_k=1^p-1 frac(p-1)!k(p-k) = pS'
          $$
          Note that $S'$ is an integer. Now
          $$
          frac(p-1)!k(p-k) equiv (k^2)^-1 bmod p
          $$
          where the inverse is taken $bmod p$. This is a consequence of Wilson’s Theorem.
          Hence
          $$
          S'equiv
          sum_k=1^p-1 (k^2)^-1 equiv
          sum_k=1^p-1 k^2 = frac(p-1)p(2(p-1)+1)6 equiv 0 bmod p
          $$
          This means that $2Sequiv 0 bmod p^2$ and so $Sequiv 0 bmod p^2$. (We need $p>3$ twice here.)






          share|cite|improve this answer











          $endgroup$



          The solution below is adapted from Notes on Wolstenholme’s Theorem by Timothy H. Choi.



          Let
          $$
          S=(p-1)!sum_k=1^p-1 frac1k
          $$
          Using your insight
          $$
          dfrac1k+dfrac1p-k=dfracpk(p-k)
          $$
          we have
          $$
          2S=(p-1)!sum_k=1^p-1 left(dfrac1k+dfrac1p-kright) =
          psum_k=1^p-1 frac(p-1)!k(p-k) = pS'
          $$
          Note that $S'$ is an integer. Now
          $$
          frac(p-1)!k(p-k) equiv (k^2)^-1 bmod p
          $$
          where the inverse is taken $bmod p$. This is a consequence of Wilson’s Theorem.
          Hence
          $$
          S'equiv
          sum_k=1^p-1 (k^2)^-1 equiv
          sum_k=1^p-1 k^2 = frac(p-1)p(2(p-1)+1)6 equiv 0 bmod p
          $$
          This means that $2Sequiv 0 bmod p^2$ and so $Sequiv 0 bmod p^2$. (We need $p>3$ twice here.)







          share|cite|improve this answer














          share|cite|improve this answer



          share|cite|improve this answer








          edited Nov 6 '13 at 12:56

























          answered Nov 6 '13 at 12:49









          lhflhf

          167k11172403




          167k11172403





















              4












              $begingroup$

              As others have noted, the congruence is not true for $p=3$, since
              $$ 2!left(1+frac 1 2right)=2+1=3,$$
              which is not divisible by $9$. We can still use what you suggested to prove the congruence holds $operatornamemod p$. Let $p$ be an odd prime, then
              beginalign*
              (p-1)!sum_k=1^p-1 frac 1 k &= (p-1)! sum_k=1^(p-1)/2 left(frac 1 k + frac 1 p-kright) \&= (p-1)!sum_k=1^(p-1)/2 fracpk(p-k) = psum_k=1^(p-1)/2 frac(p-1)!k(p-k).
              endalign*
              Since $(p-1)!$ always contains $k$ and $(p-k)$ as a factor, the fractions in the sum are integers and the result is a multiple of $p$.



              See lhf's answer for why it is even a multiple of $p^2$ as long as $p>3$.






              share|cite|improve this answer











              $endgroup$












              • $begingroup$
                The congruence will hold for any $p>3$ modulo $p^2$. That's Wolstenholme's theorem as lhf pointed out.
                $endgroup$
                – EuYu
                Nov 6 '13 at 12:39






              • 1




                $begingroup$
                I adapted my post and referencered lhf's answer, thanks for your comment!
                $endgroup$
                – Christoph
                Nov 6 '13 at 12:54















              4












              $begingroup$

              As others have noted, the congruence is not true for $p=3$, since
              $$ 2!left(1+frac 1 2right)=2+1=3,$$
              which is not divisible by $9$. We can still use what you suggested to prove the congruence holds $operatornamemod p$. Let $p$ be an odd prime, then
              beginalign*
              (p-1)!sum_k=1^p-1 frac 1 k &= (p-1)! sum_k=1^(p-1)/2 left(frac 1 k + frac 1 p-kright) \&= (p-1)!sum_k=1^(p-1)/2 fracpk(p-k) = psum_k=1^(p-1)/2 frac(p-1)!k(p-k).
              endalign*
              Since $(p-1)!$ always contains $k$ and $(p-k)$ as a factor, the fractions in the sum are integers and the result is a multiple of $p$.



              See lhf's answer for why it is even a multiple of $p^2$ as long as $p>3$.






              share|cite|improve this answer











              $endgroup$












              • $begingroup$
                The congruence will hold for any $p>3$ modulo $p^2$. That's Wolstenholme's theorem as lhf pointed out.
                $endgroup$
                – EuYu
                Nov 6 '13 at 12:39






              • 1




                $begingroup$
                I adapted my post and referencered lhf's answer, thanks for your comment!
                $endgroup$
                – Christoph
                Nov 6 '13 at 12:54













              4












              4








              4





              $begingroup$

              As others have noted, the congruence is not true for $p=3$, since
              $$ 2!left(1+frac 1 2right)=2+1=3,$$
              which is not divisible by $9$. We can still use what you suggested to prove the congruence holds $operatornamemod p$. Let $p$ be an odd prime, then
              beginalign*
              (p-1)!sum_k=1^p-1 frac 1 k &= (p-1)! sum_k=1^(p-1)/2 left(frac 1 k + frac 1 p-kright) \&= (p-1)!sum_k=1^(p-1)/2 fracpk(p-k) = psum_k=1^(p-1)/2 frac(p-1)!k(p-k).
              endalign*
              Since $(p-1)!$ always contains $k$ and $(p-k)$ as a factor, the fractions in the sum are integers and the result is a multiple of $p$.



              See lhf's answer for why it is even a multiple of $p^2$ as long as $p>3$.






              share|cite|improve this answer











              $endgroup$



              As others have noted, the congruence is not true for $p=3$, since
              $$ 2!left(1+frac 1 2right)=2+1=3,$$
              which is not divisible by $9$. We can still use what you suggested to prove the congruence holds $operatornamemod p$. Let $p$ be an odd prime, then
              beginalign*
              (p-1)!sum_k=1^p-1 frac 1 k &= (p-1)! sum_k=1^(p-1)/2 left(frac 1 k + frac 1 p-kright) \&= (p-1)!sum_k=1^(p-1)/2 fracpk(p-k) = psum_k=1^(p-1)/2 frac(p-1)!k(p-k).
              endalign*
              Since $(p-1)!$ always contains $k$ and $(p-k)$ as a factor, the fractions in the sum are integers and the result is a multiple of $p$.



              See lhf's answer for why it is even a multiple of $p^2$ as long as $p>3$.







              share|cite|improve this answer














              share|cite|improve this answer



              share|cite|improve this answer








              edited Apr 13 '17 at 12:20









              Community

              1




              1










              answered Nov 6 '13 at 12:30









              ChristophChristoph

              12.5k1642




              12.5k1642











              • $begingroup$
                The congruence will hold for any $p>3$ modulo $p^2$. That's Wolstenholme's theorem as lhf pointed out.
                $endgroup$
                – EuYu
                Nov 6 '13 at 12:39






              • 1




                $begingroup$
                I adapted my post and referencered lhf's answer, thanks for your comment!
                $endgroup$
                – Christoph
                Nov 6 '13 at 12:54
















              • $begingroup$
                The congruence will hold for any $p>3$ modulo $p^2$. That's Wolstenholme's theorem as lhf pointed out.
                $endgroup$
                – EuYu
                Nov 6 '13 at 12:39






              • 1




                $begingroup$
                I adapted my post and referencered lhf's answer, thanks for your comment!
                $endgroup$
                – Christoph
                Nov 6 '13 at 12:54















              $begingroup$
              The congruence will hold for any $p>3$ modulo $p^2$. That's Wolstenholme's theorem as lhf pointed out.
              $endgroup$
              – EuYu
              Nov 6 '13 at 12:39




              $begingroup$
              The congruence will hold for any $p>3$ modulo $p^2$. That's Wolstenholme's theorem as lhf pointed out.
              $endgroup$
              – EuYu
              Nov 6 '13 at 12:39




              1




              1




              $begingroup$
              I adapted my post and referencered lhf's answer, thanks for your comment!
              $endgroup$
              – Christoph
              Nov 6 '13 at 12:54




              $begingroup$
              I adapted my post and referencered lhf's answer, thanks for your comment!
              $endgroup$
              – Christoph
              Nov 6 '13 at 12:54











              0












              $begingroup$

              I came up with this proof while I work on this problem.



              We write $f(x) equiv_p g(x)$ if polynomials $f(x), g(x) in mathbbZ[x]$ satisfies $a_iequiv b_i$ mod $p$ for all coefficients $a_i$ of $f(x)$, and $b_i$ of $g(x)$.



              Then we have by Fermat's theorem,
              $$
              x^p-1-1 equiv_p (x-1)(x-2) cdots (x-(p-1)).
              $$



              Group the numbers in pairs as discussed here already,
              $$
              frac 11+frac 1p-1=frac p1(p-1), frac12+ frac 1p-2=frac p2(p-2), cdots, frac 1(p-1)/2+frac1(p+1)/2=frac p (p-1)(p+1)/4 .
              $$

              Each group has numerator divisible by $p$. Then it suffices to prove that the numerator $N$ in
              $$
              sum_j=1^(p-1)/2 frac 1j(p-j)=frac N(p-1)! (*)
              $$

              is divisible by $p$.



              We may write



              beginalign*x^p-1-1&equiv_p
              (x-1)(x-2)cdots (x-(p-1))\ &= prod_j=1^(p-1)/2 (x-j)(x-(p-j)) \ &=prod_j=1^(p-1)/2 (x^2-(j+(p-j))x + j(p-j))\
              &equiv_pprod_j=1^(p-1)/2 (x^2+j(p-j))
              endalign*

              Expanding the last product, we find that $N$ is the coefficient of $x^2$. Since $pgeq 5$, the coefficient of $x^2$ in the product must be divisible by $p$. Hence, $p|N$ as desired.






              share|cite|improve this answer









              $endgroup$

















                0












                $begingroup$

                I came up with this proof while I work on this problem.



                We write $f(x) equiv_p g(x)$ if polynomials $f(x), g(x) in mathbbZ[x]$ satisfies $a_iequiv b_i$ mod $p$ for all coefficients $a_i$ of $f(x)$, and $b_i$ of $g(x)$.



                Then we have by Fermat's theorem,
                $$
                x^p-1-1 equiv_p (x-1)(x-2) cdots (x-(p-1)).
                $$



                Group the numbers in pairs as discussed here already,
                $$
                frac 11+frac 1p-1=frac p1(p-1), frac12+ frac 1p-2=frac p2(p-2), cdots, frac 1(p-1)/2+frac1(p+1)/2=frac p (p-1)(p+1)/4 .
                $$

                Each group has numerator divisible by $p$. Then it suffices to prove that the numerator $N$ in
                $$
                sum_j=1^(p-1)/2 frac 1j(p-j)=frac N(p-1)! (*)
                $$

                is divisible by $p$.



                We may write



                beginalign*x^p-1-1&equiv_p
                (x-1)(x-2)cdots (x-(p-1))\ &= prod_j=1^(p-1)/2 (x-j)(x-(p-j)) \ &=prod_j=1^(p-1)/2 (x^2-(j+(p-j))x + j(p-j))\
                &equiv_pprod_j=1^(p-1)/2 (x^2+j(p-j))
                endalign*

                Expanding the last product, we find that $N$ is the coefficient of $x^2$. Since $pgeq 5$, the coefficient of $x^2$ in the product must be divisible by $p$. Hence, $p|N$ as desired.






                share|cite|improve this answer









                $endgroup$















                  0












                  0








                  0





                  $begingroup$

                  I came up with this proof while I work on this problem.



                  We write $f(x) equiv_p g(x)$ if polynomials $f(x), g(x) in mathbbZ[x]$ satisfies $a_iequiv b_i$ mod $p$ for all coefficients $a_i$ of $f(x)$, and $b_i$ of $g(x)$.



                  Then we have by Fermat's theorem,
                  $$
                  x^p-1-1 equiv_p (x-1)(x-2) cdots (x-(p-1)).
                  $$



                  Group the numbers in pairs as discussed here already,
                  $$
                  frac 11+frac 1p-1=frac p1(p-1), frac12+ frac 1p-2=frac p2(p-2), cdots, frac 1(p-1)/2+frac1(p+1)/2=frac p (p-1)(p+1)/4 .
                  $$

                  Each group has numerator divisible by $p$. Then it suffices to prove that the numerator $N$ in
                  $$
                  sum_j=1^(p-1)/2 frac 1j(p-j)=frac N(p-1)! (*)
                  $$

                  is divisible by $p$.



                  We may write



                  beginalign*x^p-1-1&equiv_p
                  (x-1)(x-2)cdots (x-(p-1))\ &= prod_j=1^(p-1)/2 (x-j)(x-(p-j)) \ &=prod_j=1^(p-1)/2 (x^2-(j+(p-j))x + j(p-j))\
                  &equiv_pprod_j=1^(p-1)/2 (x^2+j(p-j))
                  endalign*

                  Expanding the last product, we find that $N$ is the coefficient of $x^2$. Since $pgeq 5$, the coefficient of $x^2$ in the product must be divisible by $p$. Hence, $p|N$ as desired.






                  share|cite|improve this answer









                  $endgroup$



                  I came up with this proof while I work on this problem.



                  We write $f(x) equiv_p g(x)$ if polynomials $f(x), g(x) in mathbbZ[x]$ satisfies $a_iequiv b_i$ mod $p$ for all coefficients $a_i$ of $f(x)$, and $b_i$ of $g(x)$.



                  Then we have by Fermat's theorem,
                  $$
                  x^p-1-1 equiv_p (x-1)(x-2) cdots (x-(p-1)).
                  $$



                  Group the numbers in pairs as discussed here already,
                  $$
                  frac 11+frac 1p-1=frac p1(p-1), frac12+ frac 1p-2=frac p2(p-2), cdots, frac 1(p-1)/2+frac1(p+1)/2=frac p (p-1)(p+1)/4 .
                  $$

                  Each group has numerator divisible by $p$. Then it suffices to prove that the numerator $N$ in
                  $$
                  sum_j=1^(p-1)/2 frac 1j(p-j)=frac N(p-1)! (*)
                  $$

                  is divisible by $p$.



                  We may write



                  beginalign*x^p-1-1&equiv_p
                  (x-1)(x-2)cdots (x-(p-1))\ &= prod_j=1^(p-1)/2 (x-j)(x-(p-j)) \ &=prod_j=1^(p-1)/2 (x^2-(j+(p-j))x + j(p-j))\
                  &equiv_pprod_j=1^(p-1)/2 (x^2+j(p-j))
                  endalign*

                  Expanding the last product, we find that $N$ is the coefficient of $x^2$. Since $pgeq 5$, the coefficient of $x^2$ in the product must be divisible by $p$. Hence, $p|N$ as desired.







                  share|cite|improve this answer












                  share|cite|improve this answer



                  share|cite|improve this answer










                  answered Mar 27 at 22:33









                  i707107i707107

                  12.6k21647




                  12.6k21647



























                      draft saved

                      draft discarded
















































                      Thanks for contributing an answer to Mathematics Stack Exchange!


                      • Please be sure to answer the question. Provide details and share your research!

                      But avoid


                      • Asking for help, clarification, or responding to other answers.

                      • Making statements based on opinion; back them up with references or personal experience.

                      Use MathJax to format equations. MathJax reference.


                      To learn more, see our tips on writing great answers.




                      draft saved


                      draft discarded














                      StackExchange.ready(
                      function ()
                      StackExchange.openid.initPostLogin('.new-post-login', 'https%3a%2f%2fmath.stackexchange.com%2fquestions%2f554261%2fhow-prove-this-p-1-left1-frac12-frac13-cdots-frac1p-1-right%23new-answer', 'question_page');

                      );

                      Post as a guest















                      Required, but never shown





















































                      Required, but never shown














                      Required, but never shown












                      Required, but never shown







                      Required, but never shown

































                      Required, but never shown














                      Required, but never shown












                      Required, but never shown







                      Required, but never shown







                      Popular posts from this blog

                      Triangular numbers and gcdProving sum of a set is $0 pmod n$ if $n$ is odd, or $fracn2 pmod n$ if $n$ is even?Is greatest common divisor of two numbers really their smallest linear combination?GCD, LCM RelationshipProve a set of nonnegative integers with greatest common divisor 1 and closed under addition has all but finite many nonnegative integers.all pairs of a and b in an equation containing gcdTriangular Numbers Modulo $k$ - Hit All Values?Understanding the Existence and Uniqueness of the GCDGCD and LCM with logical symbolsThe greatest common divisor of two positive integers less than 100 is equal to 3. Their least common multiple is twelve times one of the integers.Suppose that for all integers $x$, $x|a$ and $x|b$ if and only if $x|c$. Then $c = gcd(a,b)$Which is the gcd of 2 numbers which are multiplied and the result is 600000?

                      Ingelân Ynhâld Etymology | Geografy | Skiednis | Polityk en bestjoer | Ekonomy | Demografy | Kultuer | Klimaat | Sjoch ek | Keppelings om utens | Boarnen, noaten en referinsjes Navigaasjemenuwww.gov.ukOffisjele webside fan it regear fan it Feriene KeninkrykOffisjele webside fan it Britske FerkearsburoNederlânsktalige ynformaasje fan it Britske FerkearsburoOffisjele webside fan English Heritage, de organisaasje dy't him ynset foar it behâld fan it Ingelske kultuergoedYnwennertallen fan alle Britske stêden út 'e folkstelling fan 2011Notes en References, op dizze sideEngland

                      Հադիս Բովանդակություն Անվանում և նշանակություն | Դասակարգում | Աղբյուրներ | Նավարկման ցանկ